find the value of the derivative (if it exists) at
each indicated extremum.

Find The Value Of The Derivative (if It Exists) Ateach Indicated Extremum.

Answers

Answer 1

Answer:

The value of the derivative at (0, 0) is zero.

Step-by-step explanation:

Given function:

[tex]f(x)=\dfrac{x^2}{x^2+4}[/tex]

To differentiate the given function, use the quotient rule and the power rule of differentiation.

[tex]\boxed{\begin{minipage}{5.4 cm}\underline{Quotient Rule of Differentiation}\\\\If $y=\dfrac{u}{v}$ then:\\\\$\dfrac{\text{d}y}{\text{d}x}=\dfrac{v \dfrac{\text{d}u}{\text{d}x}-u\dfrac{\text{d}v}{\text{d}x}}{v^2}$\\\end{minipage}}[/tex]

[tex]\boxed{\begin{minipage}{5.4 cm}\underline{Power Rule of Differentiation}\\\\If $y=x^n$, then $\dfrac{\text{d}y}{\text{d}x}=nx^{n-1}$\\\end{minipage}}[/tex]

[tex]\boxed{\begin{minipage}{5.4cm}\underline{Differentiating a constant}\\\\If $y=a$, then $\dfrac{\text{d}y}{\text{d}x}=0$\\\end{minipage}}[/tex]

[tex]\begin{aligned}\textsf{Let}\;u &= x^2& \implies \dfrac{\text{d}u}{\text{d}{x}} &=2 \cdot x^{(2-1)}=2x\\\\\textsf{Let}\;v &=x^2+4& \implies \dfrac{\text{d}v}{\text{d}{x}} &=2 \cdot x^{(2-1)}+0=2x\end{aligned}[/tex]

Apply the quotient rule:

[tex]\implies f'(x)=\dfrac{v \dfrac{\text{d}u}{\text{d}x}-u\dfrac{\text{d}v}{\text{d}x}}{v^2}[/tex]

[tex]\implies f'(x)=\dfrac{(x^2+4) \cdot 2x-x^2 \cdot 2x}{(x^2+4)^2}[/tex]

[tex]\implies f'(x)=\dfrac{2x(x^2+4)-2x^3}{(x^2+4)^2}[/tex]

[tex]\implies f'(x)=\dfrac{2x^3+8x-2x^3}{(x^2+4)^2}[/tex]

[tex]\implies f'(x)=\dfrac{8x}{(x^2+4)^2}[/tex]

An extremum is a point where a function has a maximum or minimum value. From inspection of the given graph, the minimum point of the function is (0, 0).

To determine the value of the derivative at the minimum point, substitute x = 0 into the differentiated function.

[tex]\begin{aligned}\implies f'(0)&=\dfrac{8(0)}{((0)^2+4)^2}\\\\&=\dfrac{0}{(0+4)^2}\\\\&=\dfrac{0}{(4)^2}\\\\&=\dfrac{0}{16}\\\\&=0 \end{aligned}[/tex]

Therefore, the value of the derivative at (0, 0) is zero.


Related Questions

could anyone help me out with this? thank you much in advance

Answers

Once we have this data, we can substitute the values into the formula to find the empirical probability that a person prefers apple pie given that they prefer whipped cream. Therefore, the missing probability is 1/12, and we know this because it is the value that makes the sum of all probabilities equal to 1.

What is probability?

Probability is a measure of the likelihood of an event occurring. It is expressed as a number between 0 and 1, where 0 represents an event that is impossible, and 1 represents an event that is certain to occur. For example, if the probability of winning a coin toss is 1/2, this means that there is an equal chance of the coin landing heads or tails. Probability can be calculated by dividing the number of favorable outcomes by the total number of possible outcomes. This is known as the classical probability approach. Another approach is empirical probability, where probabilities are calculated based on observed data or experiments. Lastly, subjective probability involves making an informed guess or estimate about the likelihood of an event occurring based on subjective factors such as experience, intuition, or expert opinion. Probability is a fundamental concept in statistics and is used in many application

Here,

1. The formula needed to calculate the empirical probability that a person prefers apple pie given that they prefer whipped cream is:

P(Apple Pie | Whipped Cream) = P(Apple Pie and Whipped Cream) / P(Whipped Cream)

where P(Apple Pie and Whipped Cream) is the probability that a person prefers both apple pie and whipped cream, and P(Whipped Cream) is the probability that a person prefers whipped cream.

This formula is used because it is a conditional probability, which is a measure of the probability of an event occurring given that another event has occurred. In this case, we want to find the probability that a person prefers apple pie given that they already prefer whipped cream.

To calculate P(Apple Pie and Whipped Cream), we would need to gather data on the number of people who prefer both apple pie and whipped cream. Similarly, to calculate P(Whipped Cream), we would need to gather data on the number of people who prefer whipped cream.

2. To find the missing probability, we need to use the fact that the sum of all probabilities in a probability distribution must be equal to 1. Therefore, we can set up an equation to solve for the missing probability:

1/6 + 1/3 + x + 5/12 = 1

Simplifying the equation by finding a common denominator gives:

2/12 + 4/12 + x + 5/12 = 1

Combining like terms gives:

11/12 + x = 1

Subtracting 11/12 from both sides gives:

x = 1 - 11/12

x = 1/12

To know more about probability,

https://brainly.com/question/30034780

#SPJ1

The Khan Shatyr Entertainment Center in Kazakhstan is the largest tent in the world. The spire on top is 60 m in length. The distance from the center of the tent to the outer edge is 97.5 m. The angle between the ground and the side of the tent is 42.7°.

Find the total height of the tent (h), including the spire.
Find the length of the side of the tent (x)

Answers

i. The total height of the tent including the spire is 150 m.

ii. The length of the side of the tent  x is 132.7 m.

What is a trigonometric function?

Trigonometric functions are required functions in determining either the unknown angle of length of the sides of a triangle.

Considering the given question, we have;

a. To determine the total height of the tent, let its height from the ground to the top of the tent be represented by x. Then:

Tan θ = opposite/ adjacent

Tan 42.7 = h/ 97.5

h = 0.9228*97.5

  = 89.97

h = 90 m

The total height of the tent including the spire = 90 + 60

                                           = 150 m

b. To determine the length of the side of the tent x, we have:

Cos θ = adjacent/ hypotenuse

Cos 42.7 = 97.5/ x

x = 97.5/ 0.7349

  = 132.67

The length of the side of the tent x is 132.7 m.

Learn more about the trigonometric functions at https://brainly.com/question/30860427

#SPJ1

Question 15 (2 points)
A standard deck of cards contains 4 suits of the same 13 cards. The contents of a
standard deck are shown below:

Standard deck of 52 cards
4 suits (CLUBS SPADES, HEARTS, DIAMONDS)
13 CLUBS
13 SPADES
13 HEARTS
DIAMONDS

If a card is drawn at random from the deck, what is the probability it is a jack or ten?

0
4/52- 1/13
8/52 = 2/13
48/52- 12/13

Answers

Answer: 2/13

Step-by-step explanation:

There are four jacks and four tens in a standard deck of 52 cards. However, the jack of spades and the ten of spades are counted twice since they are both a jack and a ten. Therefore, there are 8 cards that are either a jack or a ten, and the probability of drawing one of these cards at random is:

P(Jack or Ten) = 8/52 = 2/13

So the answer is 2/13.

Step-by-step explanation:

a probability is airways the ratio

desired cases / totally possible cases

in each of the 4 suits there is one Jack and one 10.

that means in the whole deck of cards we have

4×2 = 8 desired cases.

the totally possible cases are the whole deck = 52.

so, the probability to draw a Jack or a Ten is

8/52 = 2/13

Help please! I have no idea!!!! PLEASEEE

Answers

The the inverse of [tex]n = \frac{3t+8}{5}$[/tex] is [tex]t = \frac{5n-8}{3}$[/tex].

How to find inverse of the function?

To find the inverse of [tex]n = \frac{3t+8}{5}$[/tex], we need to solve for t in terms of n.

Starting with the given equation, we can first multiply both sides by 5 to get rid of the fraction:

[tex]$$5n = 3t + 8$$[/tex]

Next, we can isolate t by subtracting 8 from both sides and then dividing by 3:

[tex]$\begin{align*}5n - 8 &= 3t \\frac{5n-8}{3} &= t\end{align*}[/tex]

Therefore, the inverse of n is:

[tex]$t = \frac{5n-8}{3}$$[/tex]

We can also check that this is indeed the inverse by verifying that:

[tex]$n = \frac{3t+8}{5} = \frac{3}{5} \cdot \frac{5n-8}{3} + \frac{8}{5} = n$$[/tex]

So, the inverse of [tex]n = \frac{3t+8}{5}$[/tex] is [tex]t = \frac{5n-8}{3}$[/tex].

To know more about Inverse visit:

brainly.com/question/2541698

#SPJ1

Maximize z = 3x₁ + 5x₂
subject to: x₁ - 5x₂ ≤ 35
3x1 - 4x₂ ≤21
with. X₁ ≥ 0, X₂ ≥ 0.
use simplex method to solve it and find the maximum value​

Answers

Answer:

See below.

Step-by-step explanation:

We can solve this linear programming problem using the simplex method. We will start by converting the problem into standard form

Maximize z = 3x₁ + 5x₂ + 0s₁ + 0s₂

subject to

x₁ - 5x₂ + s₁ = 35

3x₁ - 4x₂ + s₂ = 21

x₁, x₂, s₁, s₂ ≥ 0

Next, we create the initial tableau

Basis x₁ x₂ s₁ s₂ RHS

s₁ 1 -5 1 0 35

s₂ 3 -4 0 1 21

z -3 -5 0 0 0

We can see that the initial basic variables are s₁ and s₂. We will use the simplex method to find the optimal solution.

Step 1: Choose the most negative coefficient in the bottom row as the pivot element. In this case, it is -5 in the x₂ column.

Basis x₁ x₂ s₁ s₂ RHS

s₁ 1 -5 1 0 35

s₂ 3 -4 0 1 21

z -3 -5 0 0 0

Step 2: Find the row in which the pivot element creates a positive quotient when each element in that row is divided by the pivot element. In this case, we need to find the minimum positive quotient of (35/5) and (21/4). The minimum is (21/4), so we use the second row as the pivot row.

Basis x₁ x₂ s₁ s₂ RHS

s₁ 4/5 0 1/5 1 28/5

x₂ -3/4 1 0 -1/4 -21/4

z 39/4 0 15/4 3/4 105

Step 3: Use row operations to create zeros in the x₂ column.

Basis x₁ x₂ s₁ s₂ RHS

s₁ 1 0 1/4 7/20 49/10

x₂ 0 1 3/16 -1/16 -21/16

z 0 0 39/4 21/4 525/4

The optimal solution is x₁ = 49/10, x₂ = 21/16, and z = 525/4.

Therefore, the maximum value of z is 525/4, which occurs when x₁ = 49/10 and x₂ = 21/16.

In the given figure, arrange the sides of ∆ from shortest to longest.

Answers

the sides from shortest to longest in the triangle is DE,DF and EF.

define triangle

A triangle is defined as a two-dimensional geometric shape that has three straight sides and three angles. The three sides of a triangle are referred edges or legs, and the three angles are formed where the edges or legs meet.

∠DEF=58°

∠DFE=180°-147°=33°

Angle sum of the triangle is 180°

∠DEF+∠DFE+∠EDF=180°

∠EDF=180°-33°-58°

=89°

The greatest side and the largest angle of a triangle are opposite one another, as are the shortest side and the smallest angle.

Increasing order of angle

∠DFE,∠DEF,∠EDF

Arranging the sides from shortest to longest DE,DF and EF.

to know more about angles, visit:

https://brainly.com/question/28451077

#SPJ1

Can you guys help me?​

Answers

Answer:

[tex]{ \sf{a = \frac{0.012}{0.633 -0.063 } }} \\ \\ { \sf{a = \frac{0.012}{0.57} }} \\ \\ { \sf{a = 0.021 \: (2 \: s.f)}}[/tex]

Find the unknown side lengths in similar triangles PQR and ABC.
I need an explanation on how to get the answer

Answers

Answer:

a=18 b=24

Step-by-step explanation:

We know that BC=25 and QR=30, the key term is that they are similar triangles. Therefore, BC: QR=25:30=5:6. Then BA:A=5:6=15:X

x=a=18

20:b=5:6

b=24

4.33. Find the moment-generating function of the continuous random variable
X
whose probability density is given by
f(x)={ 1
0

for 0 elsewhere ​
and use it to find
μ 1


,μ 2


, and
σ 2
.

Answers

The moment-generating function of the continuous random variable X whose probability density is given by f(x) = 1 for 0 elsewhere is M(t) = 1, and its first and second central moments, μ1′ and μ2′, and the variance, σ2, are 0, 0 and 0 respectively.

The moment-generating function of the continuous random variable X whose probability density is given by f(x) = 1 for 0 elsewhere is M(t) = 1.

Using M(t) we can calculate the first and second central moments, μ1′ and μ2′, and the variance, σ2, as follows:

μ1′ = M′(t) = 0

μ2′ = M′′(t) = 0

σ2 = μ2′ - (μ1′)2 = 0 - (0)2 = 0.

Therefore, the first and second central moments, μ1′ and μ2′, and the variance, σ2, of the continuous random variable X with probability density f(x) = 1 for 0 elsewhere are 0, 0 and 0 respectively.

Learn more about probability here:

https://brainly.com/question/30034780

#SPJ4

FILL IN THE BLANK.Given any z-score, it is safe to say that the absolute value is a good indicator of ______ and the sign (+ or -) is a good indicator of ______.

Answers

Given any z-score, it is safe to say that the absolute value is a good indicator of standard deviations away from the mean a data point is, and the sign (+ or -) is a good indicator of the data point is above or below the mean.

The z-score, also known as the standard score, is a measure of the number of standard deviations a data point is away from the mean of a distribution.

A positive z-score indicates that the data point is above the mean, while a negative z-score indicates that the data point is below the mean. The absolute value of the z-score tells us the distance of the data point from the mean in terms of the number of standard deviations.

For example, if a data point has a z-score of +2.5, we know that it is 2.5 standard deviations above the mean. If a data point has a z-score of -1.8, we know that it is 1.8 standard deviations below the mean.

The sign of the z-score is particularly useful in interpreting the direction of the deviation from the mean, while the absolute value is useful in determining the magnitude of the deviation.

To learn more about z-score click on,

https://brainly.com/question/30725725

#SPJ4

find the sum of the series 1 12 13 14 16 18 19 112 where the terms are reciprocals of the positive integers whose only prime factors are 2s and 3s.

Answers

the sum of the series is 8/3. The series consists of reciprocals of positive integers whose only prime factors are 2s and 3s.

In other words, each term of the series can be expressed as a fraction of the form 1/n, where n is a positive integer that can be factored into only 2s and 3s. For example, the first term of the series is 1/1, the second term is 1/2, and the fourth term is 1/4.

To find the sum of the series, we can first list out the terms and their corresponding values:

1/1 = 1

1/2 = 0.5

1/3 = 0.333...

1/4 = 0.25

1/6 = 0.166...

1/8 = 0.125

1/9 = 0.111...

1/12 = 0.083...

and so on.

We can see that the terms of the series decrease in value as n increases, so we can use this fact to estimate the sum of the series. For example, we can take the sum of the first few terms to get an idea of how large the sum might be:

1 + 0.5 + 0.333... + 0.25 = 2.083...

We can see that the sum is greater than 2, but less than 3. To get a more accurate estimate, we can add a few more terms:

2.083... + 0.166... + 0.125 + 0.111... = 2.486...

We can continue adding terms in this way to get a more and more accurate estimate of the sum. However, it is not easy to find a closed-form expression for the sum of the series.

Alternatively, we can use a formula for the sum of a geometric series to find the sum of the series. A geometric series is a series of the form a + ar + ar^2 + ... + ar^n, where a is the first term and r is the common ratio between terms. In our series, the first term is 1 and the common ratio is 1/2 or 1/3, depending on whether n is even or odd. Therefore, we can split the series into two separate geometric series:

1 + 1/2 + 1/8 + 1/32 + ... = 1/(1 - 1/2) = 2

1/3 + 1/12 + 1/48 + 1/192 + ... = (1/3)/(1 - 1/2) = 2/3

The sum of the two geometric series is the sum of the original series:

2 + 2/3 = 8/3

Therefore, the sum of the series is 8/3.

To know more about geometric series click here:

brainly.com/question/21087466

#SPJ4

1. Ferris Wheel Problem As you ride the Ferris wheel, your distance from the
ground varies sinusoidally with time. When the last seat1 is filled and the Ferris
wheel starts, your seat is at the position shown in the figure below. Lett be the
number of seconds that have elapsed since the Ferris wheel started. You find that
it takes you 3 seconds to reach the top, 43 feet above ground, and that the wheel
makes a
a. Sketch a graph of this sinusoidal function.
b. What is the lowest you go as the Ferris
wheel turns?
c. Find an equation of this sinusoid.
d. Predict your height above ground when
you have been riding for 4 seconds.
e. Using Desmos, find the first three times you are 18
feet above ground.
Seat
QI
Rotation
Ground

Answers

The graph of this sinusoidal function can be drawn as shown in the diagram below. As the Ferris wheel rotates, the position of the seat varies sinusoidally with respect to time.

What is graph?

Graph is a type of diagram used to represent information using a network of points and lines that connect them. It is a powerful data visualization tool that can help to effectively convey information and make relationships between data sets easier to understand. Graphs can be used to represent a wide variety of data types such as numerical, categorical or time-series data. Graphs are commonly used in mathematics, physics, biology, engineering, economics, and other disciplines.

b. The lowest point the seat reaches is 0 feet above ground, as the Ferris wheel makes a full rotation.

c. An equation of this sinusoid can be written as y = A sin (Bt + C), where A is the amplitude, B is the angular frequency, t is time, and C is the phase shift.

d. When you have been riding for 4 seconds, your height above ground is 43 feet.

e. Using Desmos, the first three times your height is 18 feet above ground can be found by solving the equation y = 18. The solutions are t = 0.715 seconds, t = 4.715 seconds, and t = 8.715 seconds.

To know more about graph click-
https://brainly.com/question/19040584
#SPJ1

for autonomous equations, find the equilibria, sketch a phase portrait, state the stability of the equilibria.

Answers

Understanding the equilibria, sketching a phase portrait, and determining the stability of equilibria for autonomous equations are important tools for analyzing and understanding the behavior of systems over time.

Autonomous equations are differential equations that do not depend explicitly on time. To find the equilibria of an autonomous equation, we set the derivative of the function to zero and solve for the values of the independent variable that satisfy the equation. These values represent points at which the function does not change over time and are known as equilibrium points.

To sketch a phase portrait for an autonomous equation, we plot the slope field of the function and then draw solutions through each equilibrium point. The resulting graph shows the behavior of the function over time and helps us understand how the solutions behave near each equilibrium point.

The stability of an equilibrium point is determined by examining the behavior of nearby solutions. If nearby solutions move toward the equilibrium point over time, the equilibrium point is stable. If nearby solutions move away from the equilibrium point over time, the equilibrium point is unstable. Finally, if the behavior of nearby solutions is inconclusive, further analysis is needed.

Here is the sketch for [tex]dx/dt = x - x^3[/tex]

       / <--- (-∞)  x=-1  (+∞) ---> \

      /                                \

  <--0-->       x=-1       x=1        0-->

      \                                /

       \ <--- (-∞)  x=1   (+∞) --->  /

Learn more about equilibria here https://brainly.com/question/29313546

#SPJ4

The Butler family and the Phillips family each used their sprinklers last summer. The water output rate for the Butler family's sprinkler was 25 L per hour. The water output rate for the Phillips family's sprinkler was 40 L per hour. The families used their sprinklers for a combined total of 55 hours, resulting in a total water output of 1750 L. How long was each sprinkler used?

Answers

The Butler family used their sprinkler for 30 hours and the Phillips family used their sprinkler for 25 hours.

Let's solve the problem with algebra.

Let x represent the number of hours the Butlers used their sprinkler, and y represent the number of hours the Phillips family used their sprinkler. We are aware of the following:

The Butler family's sprinkler had a water output rate of 25 L per hour, so the total amount of water they used is 25x.

The Phillips family's sprinkler had a water output rate of 40 L per hour, so the total amount of water they used was 40y.

The sprinklers were used by the families for a total of 55 hours, so x + y = 55.

The total amount of water produced was 1750 L, so 25x + 40y = 1750.

Using these equations, we can now solve for x and y.

First, we can solve for one of the variables in terms of the other using the equation x + y = 55. For instance, we can solve for x as follows:

x = 55 - y

When we plug this into the second equation, we get:

25(55 - y) + 40y = 1750

We get the following results when we expand and simplify:

1375 - 25y + 40y = 1750

15y = 375

y = 25

As a result, the Phillips family ran their sprinkler for 25 hours. We get the following when we plug this into the equation x + y = 55:

x + 25 = 55

x = 30

As a result, the Butlers used their sprinkler for 30 hours.

As a result, the Butler family sprinkled for 30 hours and the Phillips family sprinkled for 25 hours.

To know more similar question visit:

https://brainly.com/question/18089455

#SPJ1

Refer to the figure to the right.
(a) How many inches will the weight in the figure rise if the
pulley is rotated through an angle of 74° 50°?
(b) Through what angle, to the nearest minute, must the
pulley be rotated to raise the weight 5 in.?
9.61 in
(a) The weight in the figure will rise inches.
(Do not round until the final answer. Then round to the nearest tenth as needed.)

Answers

a) The weight will rise about 13.142 inches if the pulley is rotated through an angle of 77° 50'.

b) So, to the nearest minute, the pulley must be rotated through an angle of 23° 40' to raise the weight 4 inches.

What is angle of rotation?

In geometry, the angle of rotation refers to the amount of rotation of a geometric figure about a fixed point, usually the origin. It is the measure of the amount of rotation in degrees or radians.

Depending on the direction of rotation, the angle of rotation can be positive or negative. A positive angle of rotation represents a counterclockwise rotation, while a negative angle of rotation represents a clockwise rotation.

(a) To find out how many inches the weight will rise if the pulley is rotated through an angle of 77° 50', we need to use the formula for arc length:

arc length = r × θ

where r is the radius of the pulley, and θ is the angle of rotation in radians. To convert 77° 50' to radians, we need to convert the degrees to radians and add the minutes as a fraction of a degree:

θ = (77 + 50/60) × π/180

= 1.358 rad

Substituting r = 9.67 inches and θ = 1.358 rad into the formula for arc length, we get:

arc length = 9.67 × 1.358

= 13.142 in (approx)

(b) To find out through what angle the pulley must be rotated to raise the weight 4 inches, we can rearrange the formula for arc length to solve for θ:

θ = arc length / r

Substituting arc length = 4 inches and r = 9.67 inches, we get:

θ = 4 / 9.67

= 0.413 radians

To convert this to degrees and minutes, we can multiply by 180/π and convert the decimal part to minutes:

θ = 0.413 × 180/π

= 23.66°

To know more about geometry, visit:

https://brainly.com/question/16836548

#SPJ1

let be a geometric sequence with and ratio . for how many is it true that the smallest such that is ?

Answers

The smallest integer n such that a_n < 1 is n = -2.

Let the common ratio of the geometric progression be denoted by r. Then we have

a_2 = a_1 × r

a_3 = a_2 × r = a_1 × r^2

a_4 = a_3 × r = a_1 × r^3

a_5 = a_4 × r = a_1 × r^4

So in general, we have

a_n = a_1 × r^(n-1)

Now, we can use the given equation

(a_1357)^3 = a_34

Substituting the expressions above for a_34 and a_1357, we get

(a_1 × r^33)^3 = a_1 × r^3

Simplifying this equation by dividing both sides by a_1×r^3 and taking the cube root, we get

r^10 = 1/ (a_1^2)

Now, we need to find the smallest integer n such that a_n < 1. Using the expression for a_n above, we get

a_n < 1

a_1 × r^(n-1) < 1

r^(n-1) < 1/a_1

Taking the logarithm of both sides (with base r), we get

n-1 < log_r (1/a_1)

n < log_r (1/a_1) + 1

We know that r^10 = 1/ (a_1^2), so

1/a_1 = r^(10/2) = r^5

Substituting this into the expression above for n, we get

n < log_r (1/r^5) + 1

n < -5 + 1

n < -4

Since n is an integer, the smallest possible value for n is -3. However, this does not make sense since we cannot have a negative index for a term in the geometric progression. Therefore, the smallest integer n such that a_n < 1 is n = -2.

To verify this, we can substitute n = -2 into the expression for a_n and see if it is less than 1

a_n = a_1 × r^(n-1)

a_{-2} = a_1 × r^(-3)

Since a_1 > 1, we just need to show that r^3 > 1 to prove that a_{-2} < 1. From the equation r^10 = 1/ (a_1^2), we have

r^3 = (r^10)^(3/10) = (1/a_1^2)^(3/10) > 1

Learn more about geometric progression here

brainly.com/question/4853032

#SPJ4

The given question is incomplete, the complete question is:

Let a_1, a_2, a_3, a_4, a_5, . . . be a geometric progression with positive ratio such that a_1 > 1 and

(a_1357)^3 = a_34. Find the smallest integer n such that a_n < 1.

PLEASE HELP FAST!!
Find the slope of a line perpendicular to the line whose equation is
4x−6y=−24. Fully simplify your answer.

Answers

Answer: -3/2

Step-by-step explanation:

FIrst rearrange the equation in y = mx + b form.

4x - 6y = -24

-6y = -4x - 24

y = 2/3x + 4

If the line is perpendicular, the slope must be the negative reciprocal of the current line.

The negative reciprocal of 2/3 is -3/2.

9x+6=24
8x-4=28
-18-x=57
-4-8x=8
3x+0.7=4

Answers

Answer:

Step-by-step explanation:

To solve each of these equations, we need to isolate the variable (x) on one side of the equation. Here are the steps to solve each equation:

9x + 6 = 24

Subtract 6 from both sides:

9x = 18

Divide both sides by 9:

x = 2

Therefore, the solution to the equation is x = 2.

8x - 4 = 28

Add 4 to both sides:

8x = 32

Divide both sides by 8:

x = 4

Therefore, the solution to the equation is x = 4.

-18 - x = 57

Add 18 to both sides:

-x = 75

Multiply both sides by -1:

x = -75

Therefore, the solution to the equation is x = -75.

-4 - 8x = 8

Add 4 to both sides:

-8x = 12

Divide both sides by -8:

x = -1.5

Therefore, the solution to the equation is x = -1.5.

3x + 0.7 = 4

Subtract 0.7 from both sides:

3x = 3.3

Divide both sides by 3:

x = 1.1

Therefore, the solution to the equation is x = 1.1.

What is the equation of the line graphed?

Answers

The equation of given line which is graphed is  [tex]x+2=0.[/tex] By locating the slope (m) and y-intercept (b) in the graph of a line, we can define a linear function in the form y=mx+b.

What is the formula for a line on a line graph?

A straight line's graph equation can be expressed as [tex]y = m x + c[/tex]  , which consists of a term, a term, and a number. a new. to the a and the likes in the likes thes of the likes of thes of thes of thes of thes of thes of people.

A line graph is a type of graph that uses straight lines to connect the data points. A line graph can show how something changes over time or compares different situations1.

A horizontal line has the equation \(y = c\), where \(c\) is a constant. This means that the \(y\)-value of every point on the line is the same

Therefore, The set of all points (x,y) in the plane that satisfy the equation   [tex]y=f(x) y = f (x)[/tex]    is the function's graph.

Learn more about graph here:

https://brainly.com/question/23680294

#SPJ1

Which system of linear inequalities is represented by the graph?

y > x – 2 and x – 2y < 4
y > x + 2 and x + 2y < 4
y > x – 2 and x + 2y < 4
y > x – 2 and x + 2y < –4

Answers

The graph illustrates the linear inequality [tex]y > x - 2[/tex] and [tex]x - 2y < 4[/tex].

What is a good illustration of inequality?

The equation-like form of the formula 5x 4 > 2x + 3 has an arrowhead in lieu of the equals sign. That is an illustration of inequity. This shows that the left half, 5x 4, is bigger than the right part, 2x + 3. Finding the x numbers where the inequality holds true is what we are most interested in.

What justifies an inequality?

In mathematics, "inequality" means the connection between two reactions or values that is not equal to one another. As either an outcome, inequality occurs because of an imbalance.

We can see that the shaded region is above the line [tex]y = x - 2[/tex], which represents the inequality [tex]y > x - 2[/tex]. Additionally, the shaded region is below the line [tex]x - 2y = 4[/tex], which represents the inequality [tex]x - 2y < 4[/tex].

As a result, the graph's representation of a linear inequality arrangement is as follows:

[tex]y > x - 2[/tex] and [tex]x - 2y < 4[/tex]

To know more about inequality visit:

https://brainly.com/question/30231190

#SPJ1

Answer:

d

Step-by-step explanation:

Which of the following statements is true about an angle drawn in standard position?
Positive angles are measured clockwise.
The vertex of the angle is at point (1,1).
One side is always aligned with the positive y-axis.
One side is always aligned with the positive x-axis.

Answers

Answer:

Step-by-step explanation:

The statement that is true about an angle drawn in standard position is that one side is always aligned with the positive x-axis. The other side of the angle can be aligned with either the positive y-axis or the negative y-axis. The vertex of the angle does not necessarily have to be at point (1,1) and positive angles are measured counterclockwise.

Please help!

To prove the converse of the Pythagorean theorem, we can define a right triangle, [FILL WITH ANSWER], with sides a, b, and x. Then, we will show that if ​△ABC​ is a triangle with sides a, b, and c where a² + b² = c², then it is congruent to △DEF and therefore a right triangle.

By the Pythagorean theorem, because ​△DEF​ is a right triangle, a² + b² = x².

If ​​a² + b² = x² and a² + b² = c² ​​, then c² = x². Further, since sides of triangles are positive, then we can conclude that ​c = x​. Thus, the two triangles have congruent sides and are congruent.

If ​△ABC​ is congruent to a right triangle, then it must also be a right triangle.

Answers:
right triangle
[tex]a^{2}[/tex] + [tex]b^{2}[/tex] = [tex]x^{2}[/tex]
[tex]a^{2}[/tex] + [tex]b^{2}[/tex] = [tex]c^{2}[/tex]
​△ABC
​△DEF

Answers

If △ABC is congruent to △DEF, then it must also be a right triangle. Thus, the two triangles have congruent sides and are congruent.

what is pythagoras theorem ?

A key idea in geometry known as the Pythagorean theorem explains the relationship between the sides of a right triangle. The square of the hypotenuse, or side opposite the right angle, is said to be equal to the sum of the squares of the other two sides. It can be expressed mathematically as: a² + b² = c²

given

By defining a right triangle, DEF, with sides a, b, and x, we can demonstrate the opposite of the Pythagorean theorem. Then, we'll demonstrate that if ABC is a triangle with sides a, b, and c where a2 + b2 = c2, it is congruent to DEF and is thus a right triangle because a2 + b2 = c2.

By the Pythagorean theorem, because △DEF is a right triangle, a² + b² = x².

When a2 + b2 = c2 and a2 + b2 = x2, c2 equals x2.

If △ABC is congruent to △DEF, then it must also be a right triangle.Thus, the two triangles have congruent sides and are congruent.

To know more about Pythagoras theorem visit :-

https://brainly.com/question/14461977

#SPJ1

If △ABC is congruent to △DEF, then it must also be a right triangle. Thus, the two triangles have congruent sides and are congruent.

What is Pythagoras theorem?

A key idea in geometry known as the Pythagorean theorem explains the relationship between the sides of a right triangle. The square of the hypotenuse, or side opposite the right angle, is said to be equal to the sum of the squares of the other two sides. It can be expressed mathematically as: a² + b² = c²

By defining a right triangle, DEF, with sides a, b, and x, we can demonstrate the opposite of the Pythagorean theorem. Then, we'll demonstrate that if ABC is a triangle with sides a, b, and c where [tex]a^2 + b^2 = c^2[/tex], it is congruent to DEF and is thus a right triangle because a2 + b2 = c2.

By the Pythagorean theorem, because △DEF is a right triangle, a² + b² = x².

When[tex]a^2 + b^2 = c^2[/tex] and [tex]a^2 + b^2 = x^2[/tex], [tex]c^2[/tex] equals [tex]x^2[/tex].

If △ABC is congruent to △DEF, then it must also be a right triangle. Thus, the two triangles have congruent sides and are congruent.

To know more about Pythagoras theorem visit :-

brainly.com/question/14461977

#SPJ1

slope of secant line=?
slope of secant line=?
slope of tangent line=?
y=?

Answers

Therefore, the equation of the tangent line at (5,f(5)) is y = 18x - 65.

What is slope?

In mathematics, the slope of a line is a measure of its steepness or incline, usually denoted by the letter m. It describes the rate of change of a line in the vertical direction compared to the horizontal direction. The slope of a line can be positive, negative, zero, or undefined, depending on the angle it makes with the horizontal axis. The slope of a line is commonly calculated as the ratio of the change in the y-coordinates to the change in the x-coordinates between any two points on the line.

Here,

(A) The slope of the secant line joining (2,f(2)) and (7,f(7)) is given by:

slope = (f(7) - f(2)) / (7 - 2)

We can find f(7) and f(2) by substituting 7 and 2, respectively, into the function f(x):

f(7) = 7² + 8(7) = 49 + 56 = 105

f(2) = 2² + 8(2) = 4 + 16 = 20

Substituting these values into the formula for the slope of the secant line, we get:

slope = (105 - 20) / (7 - 2) = 85 / 5 = 17

Therefore, the slope of the secant line joining (2,f(2)) and (7,f(7)) is 17.

(B) The slope of the secant line joining (5,f(5)) and (5+h,f(5+h)) is given by:

slope = (f(5+h) - f(5)) / (5+h - 5) = (f(5+h) - f(5)) / h

We can find f(5) and f(5+h) by substituting 5 and 5+h, respectively, into the function f(x):

f(5) = 5² + 8(5) = 25 + 40 = 65

f(5+h) = (5+h)² + 8(5+h) = 25 + 10h + h² + 40 + 8h = h² + 18h + 65

Substituting these values into the formula for the slope of the secant line, we get:

slope = ((h² + 18h + 65) - 65) / h = h² / h + 18h / h = h + 18

Therefore, the slope of the secant line joining (5,f(5)) and (5+h,f(5+h)) is h+18.

(C) The slope of the tangent line at (5,f(5)) is equal to the derivative of the function f(x) at x=5. We can find the derivative of f(x) as follows:

f(x) = x² + 8x

f'(x) = 2x + 8

Substituting x=5, we get:

f'(5) = 2(5) + 8 = 18

Therefore, the slope of the tangent line at (5,f(5)) is 18.

(D) The equation of the tangent line at (5,f(5)) can be written in point-slope form as:

y - f(5) = m(x - 5)

where m is the slope of the tangent line, which we found to be 18. Substituting the values of m and f(5), we get:

y - 65 = 18(x - 5)

Simplifying, we get:

y = 18x - 65

To know more about slope,

https://brainly.com/question/30088055

#SPJ1

Use Lagrange multipliers to find the indicated extrema, assuming that x and y are positive. Minimize f(x, y) = x2 + y2 Constraint: x + 2y − 10 = 0

Answers

The value after minimizing f(x, y) = x2 + y2 with respect to constraint - x + 2y − 10 = 0, using Lagrange multipliers, is 50.

To solve this problem using Lagrange multipliers, we first write the function to be minimized as:

f(x,y) = x² + y²

And the constraint equation as:

g(x,y) = x + 2y - 10 = 0

We then form the Lagrangian function L(x,y,λ) as follows:

L(x,y,λ) = f(x,y) - λg(x,y)

Substituting in our expressions for f(x,y) and g(x,y), we get:

L(x,y,λ) = x² + y² - λ(x + 2y - 10)

Now, we take partial derivatives of L with respect to x, y and λ and set them equal to zero:

∂L/∂x = 2x - λ = 0 ∂L/∂y = 2y - 2λ = 0 ∂L/∂λ = x + 2y - 10 = 0

Solving these equations simultaneously gives us:

x = λ y = λ/2 x + 2y - 10 = 0

Substituting these values back into our original function f(x,y), we get:

f(5,5) = (5)² + (5)² = 50

Therefore, the minimum value of f(x,y) subject to the given constraint is 50.

To know more about Lagrange multipliers, refer:

https://brainly.com/question/17218339

#SPJ4

(b) Write 5 as a percentage.​

Answers

Answer:

5 as a percentage of 100 is 5/100 which is 5%

13/16+2 1/12+2 3/24





















































dfsklhgdfehuiorgjrgiy

Answers

Answer:

Fraction: 241/48

Improper fraction: 5 1/48

Decimal: 5.021

Step-by-step explanation:

To calculate the expression 13/16+2 1/12+2 3/24, I first converted the mixed numbers 2 1/12 and 2 3/24 to improper fractions. 2 1/12 is equal to 25/12 and 2 3/24 is equal to 51/24. Then, I added the three fractions 13/16, 25/12, and 51/24 by finding a common denominator, which in this case is 48. So, the expression becomes (39/48)+(100/48)+(102/48), which simplifies to (39+100+102)/48, which equals 241/48. Finally, I converted the improper fraction 241/48 to a mixed number, which is equal to 5 1/48.

Weekly CPU time used by an accounting firm has probability density function (measured in hours) given by f(x) = { 3/64 * x^2 (4 − x) 0 ≤ x ≤ 4 0 Otherwise } (a) Find the F(x) for weekly CPU time. (b) Find the probability that the of weekly CPU time will exceed two hours for a selected week. (c) Find the expected value and variance of weekly CPU time. (d) Find the probability that the of weekly CPU time will be within half an hour of the expected weekly CPU time. (e) The CPU time costs the firm $200 per hour. Find the expected value and variance of the weekly cost for CPU time. :-;​

Answers

The correct answers to the given questions are given below:

a. E(Y)= 2.4, Var (Y) = 0.64b. E(Y) = 480, Var(Y) = 25,600

What is CPU time?

CPU time, as opposed to elapsed time, which might include things like waiting for input/output operations or switching to low-power mode.

It is the length of time that a central processing unit was employed to process instructions from a computer program or operating system. The CPU time is expressed in seconds or clock ticks.

Thus, from the given question, the CPU time is measured and the expected value and variance of weekly CPU time are calculated (see image)

c. No, observing the above part, the weekly cost does not exceed $600 because the weekly cost for CPU time E(Y) =480

Read more about CPU time here:

https://brainly.com/question/19999569

#SPJ1

Type the correct answer in each box. Assume π = 3.14. Round your answer(s) to the nearest tenth. 90° 30° In this circle, the area of sector COD is 50.24 square units. The radius of the circle is units, and m AB is units.​

Answers

Therefore, the length of segment AB is approximately 7.4 units.

What is area?

Area is a mathematical concept that describes the size of a two-dimensional surface. It is a measure of the amount of space inside a closed shape, such as a rectangle, circle, or triangle, and is typically expressed in square units, such as square feet or square meters. The area of a shape is calculated by multiplying the length of one side or dimension by the length of another side or dimension. For example, the area of a rectangle can be found by multiplying its length by its width.

Here,

To find the radius of the circle, we can use the formula for the area of a sector:

Area of sector = (θ/360) x π x r²

where θ is the central angle of the sector in degrees, r is the radius of the circle, and π is approximately 3.14.

We're given that the area of sector COD is 50.24 square units and the central angle of the sector is 90°. So we can plug in these values and solve for r:

50.24 = (90/360) x 3.14 x r²

50.24 = 0.25 x 3.14 x r²

r² = 50.24 / (0.25 x 3.14)

r² = 201.28

r = √201.28

r ≈ 14.2

Therefore, the radius of the circle is approximately 14.2 units.

Next, we need to find the length of segment AB. Since AB is a chord of the circle, we can use the formula:

AB = 2 x r x sin(θ/2)

where θ is the central angle of the sector in degrees, r is the radius of the circle, and sin() is the sine function.

We're given that the central angle of sector COD is 30°. So we can plug in this value and the radius we found earlier to solve for AB:

AB = 2 x 14.2 x sin(30/2)

AB = 2 x 14.2 x sin(15)

AB ≈ 7.4

To know more about area,

https://brainly.com/question/22469440

#SPJ1

A TRIANGLE HAS TWO SIDES OF LENTHS 6 AND 9. WHAT VALUE COULD THE LENGTH OF THE THIRD SIDE BE

Answers

Answer:

The value could be any length between 3 and 15

Step-by-step explanation:

9 - 6 = 3

and

9 + 6 = 15

Find x, if √x +2y^2 = 15 and √4x - 4y^2=6

pls help very soon

Answers

Answer:

We have two equations:

√x +2y^2 = 15 ----(1)

√4x - 4y^2=6 ----(2)

Let's solve for x:

From (1), we have:

√x = 15 - 2y^2

Squaring both sides, we get:

x = (15 - 2y^2)^2

Expanding, we get:

x = 225 - 60y^2 + 4y^4

From (2), we have:

√4x = 6 + 4y^2

Squaring both sides, we get:

4x = (6 + 4y^2)^2

Expanding, we get:

4x = 36 + 48y^2 + 16y^4

Substituting the expression for x from equation (1), we get:

4(225 - 60y^2 + 4y^4) = 36 + 48y^2 + 16y^4

Simplifying, we get:

900 - 240y^2 + 16y^4 = 9 + 12y^2 + 4y^4

Rearranging, we get:

12y^2 - 12y^4 = 891

Dividing both sides by 12y^2, we get:

1 - y^2 = 74.25/(y^2)

Multiplying both sides by y^2, we get:

y^2 - y^4 = 74.25

Let z = y^2. Substituting, we get:

z - z^2 = 74.25

Rearranging, we get:

z^2 - z + 74.25 = 0

Using the quadratic formula, we get:

z = (1 ± √(1 - 4(1)(74.25))) / 2

z = (1 ± √(-295)) / 2

Since the square root of a negative number is not real, there are no real solutions for z, which means there are no real solutions for y and x.

Therefore, the answer is "no solution".

Other Questions
the ability to analyze a situation, interpret information, and make reasonable decisions involves _____. Because schoolchildren judge their own talents and limitations more realistically than preschoolers:a. they should decide which reading and math groups they wish to join.b. their self-esteem may suffer as they compare themselves with others.c. following a failure, they are less pessimistic about future failure.d. they are less likely to concede that they're not good at something. What is the meaning of "the homotopy classes of paths from x to x in a space X"? Experiment 1: what volume would the gas sample occupy at 50. 0 c? TRUE/FALSE. The Nielsen Company determines television ratings using randomly selected households so every home has an equal chance of being included. How do you think the cat and the bird ended up in these positions? Choose one image.From the perspective of the animal, write a paragraph telling the tale of what led up to this moment. what led to the fall of the Weimar Republic (paragraph writing) Which of the below are components that can be configured in the VPC section of the AWS management console? (Select TWO.)A. SubnetB. EBS volumesC. Elastic Load BalancerD. DNS recordsE. Endpoints Line A has a gradient of -5. Line B is perpendicular to line A. a) What are the coordinates of the y-intercept of line B? b) What is the equation of line B? S Give your answer in the form y where m and c are integers or fractions written in their simplest form. mx + c, Recommend two practical strategies that you could use to determine whether entrepreneurship would be the best career for you on each response also indicate how these strategies may assist you in ensuring that entrepreneurship is your best option Which Structure Is The Following True For? For _________, Arguments Are Substituted Exactly As Entered, Without Checking For Memory, Registers, Or Literals. a. Both Macros And Procedures b. Procedures c. Neither Macros Nor Procedures d. MacrosWhich structure is the following true for?For _________, arguments are substituted exactly asentered, without checking for Necesito ayuda, por favor 1. Cmo es que el romance, ejemplo de la poesa lrica, viene de la poesa pica en Espaa?2. Reconstruye la fbula (el relato en sus conexiones cronolgicas y causales) a partir del siuzhet (el relato tal como se presenta textualmente) en Las medias rojas, pp. 52-54 hugh, a researcher, is studying the relationship between hard work and rewards. he notices that people who work hard are given increments and rewards, which in turn makes them work harder. this type of relationship is called a Sharon and Kaylyn are playing air hockey. Sharon hits the hockey puck sending it at a velocity of 6 m/s and it hits Kaylyn's puck Both hockey pucks have a mass of 0.3 kg Sharon's puck stops after it hits Kaylyn's puck According to the law of conservation of momentum, what should the resulting velocity of Kaylyn's puck be if it. was at rest before colliding with Sharon's puck? A. Equal to the velocity of Sharon's puck before they collided B. Twice the velocity of Sharon's puck before they collided C. Half the velocity of Sharon's puck before they collidedD. Equal to the velocity of Sharon's puck ball after they collided a firm can choose a global market entry strategy from among four choices: exporting, blank , joint venture, and direct investment. multiple choice question. strategic alliance importing market penetration licensing loss-framed messages are particularly effective in promoting _____ behaviors, whereas gain-framed messages are particularly effective in promoting behaviors. a worker who works a job such as cleaning houses or child care but is not reporting the income earned to the tax authorities, would be categorized by the government as: How many proper subsets are in {2,4,6,8...100} during the process of gene expression in eukaryotes, , together with that help it find the promoter, will read the of dna in the direction. it will synthesize a in the direction. calculate T1 , T2 and T3